Yahoo Answers is shutting down on May 4th, 2021 (Eastern Time) and beginning April 20th, 2021 (Eastern Time) the Yahoo Answers website will be in read-only mode. There will be no changes to other Yahoo properties or services, or your Yahoo account. You can find more information about the Yahoo Answers shutdown and how to download your data on this help page.

Integral Test Problem?

Find the sum from n=2 to infinity of (e^(1/n))/(n^2).

I'm getting that it converges because the antiderivative is -e^(1/x) and that approaches 1 as x approaches infinity, meaning that both the improper integral and series converges; but the textbook says it diverges.

4 Answers

Relevance
  • Awms A
    Lv 7
    5 years ago
    Favorite Answer

    It converges. You can also see it by

    0 <= e^(1/n) / n^2 <= e / n^2

    and the fact the sum of e/n^2 converges by the p-series test.

    Unless there's a misunderstanding somewhere about what the problem is, I'd have to conclude the textbook is wrong in this case (mistakes happen)

  • Anonymous
    5 years ago

    74

  • Anonymous
    5 years ago

    there was clearly a mistake as sometimes that happens as you should email the textbook company as may even give you something in return

  • 5 years ago

    ehhhhh e^1/2 is something i cant remember but its multiplied by (4)

Still have questions? Get your answers by asking now.